LSAT and Law School Admissions Forum

Get expert LSAT preparation and law school admissions advice from PowerScore Test Preparation.

User avatar
 Dave Killoran
PowerScore Staff
  • PowerScore Staff
  • Posts: 5852
  • Joined: Mar 25, 2011
|
#79630
Complete Question Explanation
(The complete setup for this game can be found here: lsat/viewtopic.php?t=6157)

The correct answer choice is (C).

Answer choice (A) is incorrect because if J were connected to L and M, that would violate the fifth rule that states that J is connected to exactly two bridges (remember, J is also connected to O).

Answer choice (B) is incorrect because it violates the sixth rule that states that K connects to exactly one other island.

Answer choice (C) is the correct answer. If L connect to both J and M, then K could connect to M or O.

Answer choice (D) is incorrect because M would be connected to four islands (O, J, K, and L), a violation of the third rule.

Answer choice (E) is incorrect because O would be connected to four islands (J, M, K, and L), a violation of the third rule.

Get the most out of your LSAT Prep Plus subscription.

Analyze and track your performance with our Testing and Analytics Package.